Условие Миттаг-Леффлера как необходимое и достаточное Пусть А
1 ← A
2 ← A
3 ← … -- проективная система абелевых групп. Первый (и единственный, кроме исходного функтора) производный функтор проективного предела lim
n1 A
n проективной системы (A
n) можно посчитать как коядро гомоморфизма
1−shift: ∏
n=1∞ A
n → ∏
n=1∞ A
n.
Ядром того же гомоморфизма является собственно проективный предел lim
n A
n.
Достаточным условием зануления производного функтора lim
n1 A
n является следующее
условие Миттаг-Леффлера: если для любого m ≥ 1 убывающая последовательность подгрупп im p
m,n: A
n → A
m, n ≥ m, стабилизируется при n → ∞, то lim
n1 A
n = 0.
Является ли это условие необходимым для зануления группы lim
n1 A
n ? Нет, не является. Достаточно заметить, что производный функтор проективного предела, как и сам функтор проективного предела, коммутирует с операцией бесконечного произведения проективных систем абелевых групп.
Подобрав последовательность проективных систем абелевых групп, каждая из которых удовлетворяет условию Миттаг-Леффлера, но моменты стабилизации подгрупп-образов в каждой следующей проективной системе наступают позже, чем в предыдущей, и взяв прямое произведение такой бесконечной последовательности проективных систем, можно получить проективную систему абелевых групп, в которой стабилизация подгрупп-образов отображений проекции не наступает никогда, а производный функтор проективного предела этой проективной системы равен нулю, тем не менее.
Можно ли превратить условие Миттаг-Леффлера в необходимое и достаточное? Можно.
Теорема. Пусть A
1 ← A
2 ← A
3 ← … -- проективная система абелевых групп. Рассмотрим бесконечную прямую сумму счетного числа копий проективной системы (A
n) и обозначим эту проективную систему через (B
n) = ⊕
ω (A
n). Тогда следующие три условия эквивалентны:
- проективная система (A
n) удовлетворяет условию Миттаг-Леффлера;
- проективная система (B
n) удовлетворяет условию Миттаг-Леффлера;
- lim
n1 B
n = 0.
Кто-нибудь из читающих эти строки встречал такой результат где-нибудь?
P.S. Спросил на MathOverflow:
https://mathoverflow.net/questions/270762/the-mittag-leffler-condition-as-necessary-and-sufficient